4
$\begingroup$

The following question was closed because apparently it was very similary to a question that I asked a month or two back on this site. I thought at first that this was an exact duplicate and voted to close. However closer inspection revealed that the OP is also asking about another result concerning the fact that any finite extension of a finite field is separable - the question that I asked does not address this. In my opinion this question should be reopened. To those that voted to close, can you vote to reopen please?

Thanks.

$\textbf{Edit:}$ The question has now been reopened.

$\endgroup$
0

0

You must log in to answer this question.